Angular momentum commutation relation, extra terms?

Click For Summary
The discussion centers on the commutation relation between the x and y components of angular momentum defined as L = r x P. The user calculates the angular momentum components and finds complications when applying the commutation relation [Lx, Ly], leading to confusion over the presence of five terms instead of the expected four. It is clarified that one term can be rewritten using the product rule for differentiation, resulting in the additional terms. The conversation concludes with a suggestion that using commutators simplifies the derivation process significantly. Overall, the focus is on understanding the derivation of angular momentum commutation relations in quantum mechanics.
rwooduk
Messages
757
Reaction score
59

Homework Statement


What is the commutation relation between the x and y components of angular momentum L = r X P

Homework Equations


None.

The Attempt at a Solution


I do r X p and get the angular momentum componants:L_{x} = (-i \hbar) (y \frac{d}{dz} - z \frac{d}{dy})
L_{y} = (-i \hbar) (z \frac{d}{dx} - x \frac{d}{dz})
L_{z} = (-i \hbar) (x \frac{d}{dy} - y \frac{d}{dx})

then when I attempt to put into the commutation relation [Lx,Ly] it comes out very complicated.

My question:

I found a derivation BUT it has extra terms in it (circled in red)

9EVMLQo.jpg


Why are there 5 terms when there should only be 4 when multiplying out 2 brackets? It's important because those 2 extra terms enable the correct solution. Or have I missed something?

Thankyou for any help in advance.

Source of derivation: http://www2.ph.ed.ac.uk/~ldeldebb/docs/QM/lect8.pdf
 
Physics news on Phys.org
rwooduk said:

Homework Statement


What is the commutation relation between the x and y components of angular momentum L = r X P

Homework Equations


None.

The Attempt at a Solution


I do r X p and get the angular momentum componants:L_{x} = (-i \hbar) (y \frac{d}{dz} - z \frac{d}{dy})
L_{y} = (-i \hbar) (z \frac{d}{dx} - x \frac{d}{dz})
L_{z} = (-i \hbar) (x \frac{d}{dy} - y \frac{d}{dx})

then when I attempt to put into the commutation relation [Lx,Ly] it comes out very complicated.

My question:

I found a derivation BUT it has extra terms in it (circled in red)

9EVMLQo.jpg


Why are there 5 terms when there should only be 4 when multiplying out 2 brackets? It's important because those 2 extra terms enable the correct solution. Or have I missed something?

Thankyou for any help in advance.

Source of derivation: http://www2.ph.ed.ac.uk/~ldeldebb/docs/QM/lect8.pdf

There are four terms when you multiply the brackets. However, one of them can be rewritten with two terms, namely
$$
y\partial_z z \partial_x = y \partial_x + yz \partial_z \partial_x.
$$
This is just the product rule for differentiation.
 
Orodruin said:
There are four terms when you multiply the brackets. However, one of them can be rewritten with two terms, namely
$$
y\partial_z z \partial_x = y \partial_x + yz \partial_z \partial_x.
$$
This is just the product rule for differentiation.

thats great, many thanks for your help!
 
Just to complete this thread, this derivation is MUCH easier using commutators! Here's an excellent web page detailing the process:

http://physicspages.com/2011/07/19/angular-momentum-commutators/
 
Question: A clock's minute hand has length 4 and its hour hand has length 3. What is the distance between the tips at the moment when it is increasing most rapidly?(Putnam Exam Question) Answer: Making assumption that both the hands moves at constant angular velocities, the answer is ## \sqrt{7} .## But don't you think this assumption is somewhat doubtful and wrong?

Similar threads

  • · Replies 5 ·
Replies
5
Views
2K
Replies
3
Views
2K
  • · Replies 2 ·
Replies
2
Views
2K
  • · Replies 6 ·
Replies
6
Views
1K
  • · Replies 2 ·
Replies
2
Views
2K
  • · Replies 4 ·
Replies
4
Views
1K
  • · Replies 5 ·
Replies
5
Views
1K
  • · Replies 17 ·
Replies
17
Views
852
  • · Replies 4 ·
Replies
4
Views
2K
Replies
12
Views
2K